Complex Integration: Contour Evaluation and Estimation Lemma

Click For Summary
The discussion focuses on evaluating the contour integral ∫C Im(z − i)dz, where C consists of a circular arc and a line segment. Participants suggest using the parameterization z = x + iy and converting the integral into terms of t for easier calculation. For the second question regarding the circle |z| = 4, it is recommended to apply the estimation lemma to show the inequality involving the integral of ez/(z+1). The conversation emphasizes the importance of proper parameterization and substitution in contour integration. Overall, the thread provides insights into solving complex integrals using contour evaluation techniques.
nickolas2730
Messages
28
Reaction score
0
1.Evaluate ∫C Im(z − i)dz, where C is the contour consisting of the circular arc along |z| = 1 from z = 1 to z = i and the line segment from z = i to z = −1.

2. Suppose that C is the circle |z| = 4 traversed once. Show that
§C (ez/(z+1)) dz ≤ 8∏e4/3

For question 1, should i let z= x+yi to solve the question?
and it said the I am part so i just need to consider the "yi"?

i tried but really have no idea on these 2 questions..
Please help
Thanks
 
Physics news on Phys.org
Yes, let z=x+iy and then integrate iy-i over z=e^(it) as t goes from 0 to pi/2. So wouldn't that be:

\int_0^{\pi/2} Im(z-i)dz,\quad z=e^{it}

You can convert that to all in t right? dz=ie^(it)dt=i(cos(t)+isin(t))dt and won't y be just sin(t)?

For the contour from i to -1, need to do that one in terms of z=x(t)+iy(t). Isn't that line just y=x+1 as x goes from 0 to -1? So suppose I let x=x(t)=t, then y(t) is? Now substitute all that into the integral:

\int_0^{-1} (iy-i)dz,\quad z=x(t)+iy(t)

with dz=x'(t)+iy'(t)
 
For the second one, use the http://en.wikipedia.org/wiki/Estimation_lemma" .
 
Last edited by a moderator:
Question: A clock's minute hand has length 4 and its hour hand has length 3. What is the distance between the tips at the moment when it is increasing most rapidly?(Putnam Exam Question) Answer: Making assumption that both the hands moves at constant angular velocities, the answer is ## \sqrt{7} .## But don't you think this assumption is somewhat doubtful and wrong?

Similar threads

  • · Replies 3 ·
Replies
3
Views
2K
Replies
1
Views
1K
  • · Replies 2 ·
Replies
2
Views
1K
Replies
13
Views
2K
  • · Replies 8 ·
Replies
8
Views
2K
  • · Replies 17 ·
Replies
17
Views
3K
  • · Replies 2 ·
Replies
2
Views
911
Replies
32
Views
3K
  • · Replies 2 ·
Replies
2
Views
2K
Replies
8
Views
3K